Hardware Upgrade Forum

Hardware Upgrade Forum (https://www.hwupgrade.it/forum/index.php)
-   Scienza e tecnica (https://www.hwupgrade.it/forum/forumdisplay.php?f=91)
-   -   [Official Thread]Richieste d'aiuto in MATEMATICA: postate qui! (https://www.hwupgrade.it/forum/showthread.php?t=1221191)


guest84 10-11-2011 15:14

Qualcuno riuscirebbe a spiegami come una funzione possa essere derivabile ma avere derivata discontinua?

Lampo89 10-11-2011 15:56

Quote:

Originariamente inviato da guest84 (Messaggio 36324912)
Qualcuno riuscirebbe a spiegami come una funzione possa essere derivabile ma avere derivata discontinua?

La teoria che ci sta dietro è identica a quella della continuità di una funzione. Più che altro servirebbe un esempio di funzione derivabile in un punto ma discontinua in esso. Esempio classico tratto dalla perversione dei professori di analisi matematica:

f(x) = x^2 sin(1/x) per x diversi da zero
0 per x uguale a zero
definita in questo modo la funzione risulta continua su R (sin(1/x) è limitato per x->0, mentre x è infinitesimo).

La funzione è composizione di funzioni e prodotto di funzioni continue e derivabili su R escluso lo zero. Dunque la funzione sarà certamente derivabile in R escluso lo zero (attenzione: ciò non significa che la funzione non è derivabile in zero)
calcolando dunque la derivata della funzione con le regole di derivazione si ottiene
f'(x) = 2x sin(1/x) - cos(1/x) per x diversi da zero
applicando la definizione di limite per verificare la derivabilità in 0 si trova che:

lim x->0 ( f(x) - f(0) )/x = lim x->0 x sin(1/x) = 0

Ricapitolando

f'(x) = 2x sin(1/x) - cos(1/x) per x diversi da zero
0 per x = 0

perciò f(x) è derivabile in R
Inoltre f'(x) è continua in tutti in punti diversi da zero (perchè è una composizione di continue come coseno, potenze etc)
mentre invece nel punto x = 0 accade la cosa seguente:
lim x->0 f'(x) non esiste
e dunque lim x->0 f'(x) è diverso da f'(0) =0
e quindi f'(x) non è una funzione continua in 0 (possiede una discontinuità di seconda specie del tipo non esiste limite)
spero di essere stato chiaro
p.s. una funzione derivabile in un punto ma ivi discontinua non può avere una discontinuità di tipo salto perchè implicherebbe che la diversità tra derivata destra e sinistra nel punto in esame e quindi la non derivabilità; mi pare di ricordare che esiste un teorema che garantisce che le eventuali discontinuità di una funzione derivabile possono essere solo di seconda specie, dovrebbe essere un corollario al teorema di lagrange prova a googlare in giro .

guest84 10-11-2011 16:17

Quote:

Originariamente inviato da Lampo89 (Messaggio 36325252)
La teoria che ci sta dietro è identica a quella della continuità di una funzione. Più che altro servirebbe un esempio di funzione derivabile in un punto ma discontinua in esso. Esempio classico tratto dalla perversione dei professori di analisi matematica:

f(x) = x^2 sin(1/x) per x diversi da zero
0 per x uguale a zero
definita in questo modo la funzione risulta continua su R (sin(1/x) è limitato per x->0, mentre x è infinitesimo).

La funzione è composizione di funzioni e prodotto di funzioni continue e derivabili su R escluso lo zero. Dunque la funzione sarà certamente derivabile in R escluso lo zero (attenzione: ciò non significa che la funzione non è derivabile in zero)
calcolando dunque la derivata della funzione con le regole di derivazione si ottiene
f'(x) = 2x sin(1/x) - cos(1/x) per x diversi da zero
applicando la definizione di limite per verificare la derivabilità in 0 si trova che:

lim x->0 ( f(x) - f(0) )/x = lim x->0 x sin(1/x) = 0

Ricapitolando

f'(x) = 2x sin(1/x) - cos(1/x) per x diversi da zero
0 per x = 0

perciò f(x) è derivabile in R
Inoltre f'(x) è continua in tutti in punti diversi da zero (perchè è una composizione di continue come coseno, potenze etc)
mentre invece nel punto x = 0 accade la cosa seguente:
lim x->0 f'(x) non esiste
e dunque lim x->0 f'(x) è diverso da f'(0) =0
e quindi f'(x) non è una funzione continua in 0 (possiede una discontinuità di seconda specie del tipo non esiste limite)
spero di essere stato chiaro
p.s. intuitivamente mi verrebbe da dire che una funzione derivabile in un punto ma ivi discontinua non possa avere una discontinuità di tipo salto; mi pare di ricordare che esiste un teorema che garantisce che le eventuali discontinuità di una funzione derivabile possono essere solo di seconda specie, dovrebbe essere un corollario al teorema di lagrange prova a googlare in giro .

Grazie mille :)
In realtà proprio è proprio questo esempio che ha utilizzato il professore, soltanto che non l'ha spiegato bene e sul libro ti testo non c'è.

Ziosilvio 10-11-2011 16:33

Quote:

Originariamente inviato da guest84 (Messaggio 36324912)
Qualcuno riuscirebbe a spiegami come una funzione possa essere derivabile ma avere derivata discontinua?

Per se e solo se e 0 altrove, considera .

EDIT: però in effetti così la derivata non è definita in x=1 e x=2... sorry.

Lampo89 10-11-2011 16:35

Quote:

Originariamente inviato da guest84 (Messaggio 36325466)
Grazie mille :)
In realtà proprio è proprio questo esempio che ha utilizzato il professore, soltanto che non l'ha spiegato bene e sul libro ti testo non c'è.

sì quello è l'esempio tipico ... or ora mi è venuta in mente il corollario che ti dicevo, non l'ho trovato su un libro di testo (in verità è un vago ricordo dell'esame di analisi I, l'avevo letto sul libro ma non potendolo consultare sono andato a naso) ma mi sembra una dimostrazione corretta e sacrosanta. quindi lo scrivo magari per utilità tua e di altri che leggeranno

corollario a lagrange: data f(x) definita in [a,b] ed ivi continua, derivabile nell'aperto (a,b] e tale che esista finito
lim x->a f'(x) = z

(n.b. non si può dire che z = f'(a) perchè
motivazione 0: non si ipotizza che f sia derivabile in a
motivazione 1: non si ipotizza che f' sia continua in a )


sotto tali ipotesi, si dimostra che f è derivabile in a e lim x->a f'(x) = f'(a)

velocissima dimostrazione: considero un punto c interno ad [a,b]. Applico il teo di lagrange ad f considerando l'intervallo [a,c]: esiste dunque un punto interno a tale intervallo (e lo chiamo d) tale che

f'(d) = (f(c)-f(a))/(c-a)

è da notare che il punto d dipende solamente dal punto c scelto (che è arbitrario).

facendo tendere c ad a, d tende ad a (dato che d è interno all'intervallo [a,c])

dunque dalla relazione sopra si ha che:
lim d->a f'(d) = lim c->a (f(c)-f(a))/(c-a)

il secondo membro è la definizione di derivata in a , mentre il membro sx è finito e pari a z dunque
lim d->a f'(d) = lim c->a (f(c)-f(a))/(c-a) = z

perciò z = f'(a) e lim d->a f'(d) = f'(a) = z

cioè f è derivabile in a e la derivata è continua in a.

da questo corollario con un piccolo volo di fantasia viene fuori che la derivata di una funzione derivabile può avere solo discontinuità di seconda specie

Amsirak 10-11-2011 18:19

Quote:

Originariamente inviato da Ziosilvio (Messaggio 36323607)
Ma aggiungere e togliere 1 sopra e sotto, e poi dividere per x sopra e sotto, è tanto complicato? :confused:

hai ragione, era la soluzione migliore. Alla fine ho risolto dividendo il numeratore per 7^x, il denominatore per 5^x e poi dividendo numeratore e denominatore per x.

Grazie

peppozzolo 02-12-2011 08:16

ragazzi un piccolo aiutino col dominio di queste due funzioni

1)log(arcsin((x+6)/(x-2)))

2)(pigreco-arcsin(x))^1/2

ci sto sbattendo la testa da parecchio ma non riesco a venirne a capo

grazie a chi mi dara una mano ;)

Ziosilvio 02-12-2011 09:45

Quote:

Originariamente inviato da peppozzolo (Messaggio 36468929)
ragazzi un piccolo aiutino col dominio di queste due funzioni

1)log(arcsin((x+6)/(x-2)))

2)(pigreco-arcsin(x))^1/2

ci sto sbattendo la testa da parecchio ma non riesco a venirne a capo

grazie a chi mi dara una mano ;)

log x è definito solo per x > 0, quindi deve essere arcsin ((x+6)(x-2)) > 0.
arcsin x è definito solo per x tra -1 e 1 e ha il segno del suo argomento, quindi...

Eccetera...

peppozzolo 02-12-2011 09:48

grazie silvio ma il mio problema si riferiva piu che altro al risultato con il quale non mi riesco a trovare e volevo sapere proprio i singoli passaggi per magari capire dove sta l'errore di fondo :D
Grazie ancora per l'attenzione

Ziosilvio 02-12-2011 14:34

Quote:

Originariamente inviato da peppozzolo (Messaggio 36469582)
grazie silvio ma il mio problema si riferiva piu che altro al risultato con il quale non mi riesco a trovare e volevo sapere proprio i singoli passaggi per magari capire dove sta l'errore di fondo :D
Grazie ancora per l'attenzione

Dài, che il secondo è proprio facile facile...

peppozzolo 02-12-2011 15:41

per la seconda funzione il mio problema e questo
praticamente imponendo

arcsin(x)<=Pigreco

dovrebbe essere sempre verificata quindi il dominio dovrebbe ridurso a quello dell'argomento [-1,1]

ma il libro mi porta [-1,0]
quindi credo faccia il successivo passaggio
cioe
sin(arcsin(x))<=sin(Pigreco)
quindi
x<=0

In questo caso come mi devo comportare?
la seconda soluzione e quella che va semrep applicata?
ps sono riuscito a risolvere la prima :)

Lampo89 02-12-2011 20:55

Quote:

Originariamente inviato da peppozzolo (Messaggio 36472353)
per la seconda funzione il mio problema e questo
praticamente imponendo

arcsin(x)<=Pigreco

dovrebbe essere sempre verificata quindi il dominio dovrebbe ridurso a quello dell'argomento [-1,1]

ma il libro mi porta [-1,0]
quindi credo faccia il successivo passaggio
cioe
sin(arcsin(x))<=sin(Pigreco)
quindi
x<=0

In questo caso come mi devo comportare?
la seconda soluzione e quella che va semrep applicata?
ps sono riuscito a risolvere la prima :)

http://www.wolframalpha.com/input/?i...x%29+%3C%3D+Pi

direi che il libro sbaglia clamorosamente. Quello che dici è correttissimo: l'inversa del seno è definita tra -1 e 1, ed è limitata tra -pi/2 e pi/2, di conseguenza la disuguaglianza è sempre verificata. Attento però: il tuo passaggio, quello in cui applichi sin a ambo i membri per far andare via l'arcoseno è sbagliato. Prova a pensarci un attimo, in caso ti dò una dritta!

Ziosilvio 02-12-2011 22:32

Quote:

Originariamente inviato da peppozzolo (Messaggio 36472353)
per la seconda funzione il mio problema e questo
praticamente imponendo

arcsin(x)<=Pigreco

dovrebbe essere sempre verificata quindi il dominio dovrebbe ridurso a quello dell'argomento [-1,1]

Infatti.
Quote:

Originariamente inviato da peppozzolo (Messaggio 36472353)
ma il libro mi porta [-1,0]
quindi credo faccia il successivo passaggio
cioe
sin(arcsin(x))<=sin(Pigreco)

Sbagliando clamorosamente, perché arcsin porta [-1,1] in [-Pi/2,Pi/2].
Quote:

Originariamente inviato da peppozzolo (Messaggio 36472353)
In questo caso come mi devo comportare?

Lasciando perdere la parte che contiene l'errore ;)
Quote:

Originariamente inviato da peppozzolo (Messaggio 36472353)
ps sono riuscito a risolvere la prima :)

Ottimo :mano:

peppozzolo 03-12-2011 08:36

Quote:

Originariamente inviato da Lampo89 (Messaggio 36474244)
http://www.wolframalpha.com/input/?i...x%29+%3C%3D+Pi

direi che il libro sbaglia clamorosamente. Quello che dici è correttissimo: l'inversa del seno è definita tra -1 e 1, ed è limitata tra -pi/2 e pi/2, di conseguenza la disuguaglianza è sempre verificata. Attento però: il tuo passaggio, quello in cui applichi sin a ambo i membri per far andare via l'arcoseno è sbagliato. Prova a pensarci un attimo, in caso ti dò una dritta!

Vogliamo la dritta :D ,adesso non so piu in quali casi applicarlo :confused: ,solamente in disequzioni contenute nel dominio e codominio della funzione arcoseno?

ps grazie a entrambi per il supporto

Lampo89 03-12-2011 10:04

Quote:

Originariamente inviato da peppozzolo (Messaggio 36475590)
Vogliamo la dritta :D ,adesso non so piu in quali casi applicarlo :confused: ,solamente in disequzioni contenute nel dominio e codominio della funzione arcoseno?

ps grazie a entrambi per il supporto

ok vedo che ci sei :D solo per disequazioni ed equazioni in cui il membro di dx (un numero) è compreso tra -Pi/2 e Pi/2.

Helyanwe 15-12-2011 18:04

errore ... sorry :p

lauda 19-12-2011 13:56

Ciao a tutti,
Il mio è un quesito di probabilità più che di matematica, spero di non essere troppo OT:
Sono disponibili 4 borse di laurea, i candidati sono 3 donne e 7 uomini!
Le borse di studio vengono assegnate per estrazione a sorte, si calcoli la probabilità che almeno una donna riceva la borsa di studio.

Il libro calcola la probabilità che le borse vengano assegnate solo agli uomini e poi per differenza trova la probabilità che almeno una donna riceva la borsa di studio!

Ma come si fa a calcolare direttamente tale valore?

Ziosilvio 20-12-2011 10:34

Calcolare direttamente si può.
Però è scomodo, perché si tratta fondamentalmente di fare quattro estrazioni senza reinserimento, quindi calcolare le probabilità congiunte è un po' faticoso, noioso, e anche aperto agli errori.
Calcolando la probabilità complementare, invece, si fa praticamente un conto solo.

robertogl 30-12-2011 14:49

piccolo problema: dimostrazione per induzione. Devo dimostrare che:
(ab)^n = a^n per b^n
lo faccio per 0 e po devo dimostrarlo per n+1. Posso dire che (ab)^(n+1) non è altro che ab ripetuto n+1 volte, e lo stesso di a e di b a sinistra? Così posso dire che a e b si ripetono uguali volte a sinistra e destra...:fagiano:
altrimenti
(ab)^n per (ab) = a^n per a per b^n per b

semplifico a e b quindi

(ab)^n = a^n per b^n

e poi faccio (ab)^(n-1) per ab= a^(n-1) per a per b (n-1) per b

e via così.
Potrei dimostrarlo in qualche passaggio con l'esponenziale, ma penso di non poterlo fare ancora :fagiano:

Ziosilvio 30-12-2011 15:03

Per effettuare la dimostrazione per induzione, devi sfruttare l'ipotesi induttiva (a*b)^n = a^n *b^n per dimostrare il passo induttivo (a*b)^(n+1) = a^(n+1) * b^(n+1).
Quindi, un ragionamento corretto sarebbe:
(a*b)^(n+1)
= (a*b)^n * (a*b) per la definizione di potenza
= (a^n *b^n) * a * b per ipotesi indutiva
= a * (a^n *b^n) * b per le proprietà commutativa e associativa
= a^(n+1) * b^(n+1)

robertogl 30-12-2011 15:23

Quote:

Originariamente inviato da Ziosilvio (Messaggio 36639839)
Per effettuare la dimostrazione per induzione, devi sfruttare l'ipotesi induttiva (a*b)^n = a^n *b^n per dimostrare il passo induttivo (a*b)^(n+1) = a^(n+1) * b^(n+1).
Quindi, un ragionamento corretto sarebbe:
(a*b)^(n+1)
= (a*b)^n * (a*b) per la definizione di potenza
= (a^n *b^n) * a * b per ipotesi indutiva
= a * (a^n *b^n) * b per le proprietà commutativa e associativa
= a^(n+1) * b^(n+1)

non mi è chiara una cosa: io sono nel procedimento per dimostrare che (a*b)^n = a^n *b^n, l'ho fatto per 1 e adesso devo farlo per (n+1). Non posso fare il passaggio
= (a*b)^n * (a*b)
= (a^n *b^n) * a * b per ipotesi induttiva
perché è quello che devo dimostrare..o no? :stordita:

DanieleC88 30-12-2011 15:27

No, il caso base ti serve per avere un punto di partenza, e il passo induttivo ti serve per dimostrare come passare dalla formula per un generico n a quella per il successivo n+1. Se hai n=1, allora n+1=2 ed è dimostrato anche per un secondo numero, e così via fino all'infinito.

Se preferisci puoi anche assumere di aver dimostrato per n-1, e concludere il passo induttivo dimostrando per n.

robertogl 30-12-2011 15:34

Quote:

Originariamente inviato da DanieleC88 (Messaggio 36639994)
No, il caso base ti serve per avere un punto di partenza, e il passo induttivo ti serve per dimostrare come passare dalla formula per un generico n a quella per il successivo n+1. Se hai n=1, allora n+1=2 ed è dimostrato anche per un secondo numero, e così via fino all'infinito.

Se preferisci puoi anche assumere di aver dimostrato per n-1, e concludere il passo induttivo dimostrando per n.

Ok grazie, credo di aver capito :)

lauda 02-01-2012 12:26

Ciao a tutti,
Qualcuno è in grado di risolvere questo problema?

Un soggetto scelto a caso viene sottoposto al test per una malattia genetica. Si supponga che nella popolazione un individuo ogni 10000 abbia la malattia e che la probabilita che il test risulti positivo quando l’individuo è malato sia 0.99, mentre la probabilità che il test risulti positivo quando l’individuo non è malato sia 0.05. Sapendo che il soggetto è risultato positivo al test, determinare la probabilità che egli sia malato.


Grazie mille

P.s. Ho risolto, scusate per il post! Non credo si possa cancellare :|

Ziosilvio 02-01-2012 15:13

Perché invece non ci spieghi come hai fatto, in modo che chi leggerà dopo possa capire come si fanno questi esercizi? ;)

thomas27 05-01-2012 11:59

esatto! potrebbe essere utile per tutti :) a me per esempio interessa :)

luxorl 20-01-2012 10:19

Ciao a tutti, ho un piccolo dubbio sul problema seguente:



Devo calcolarmi le coordinate X dei punti A, B, C e D.

Quello che so:
  1. I punti A e C hanno la stessa coordinata X
  2. I punti B e D hanno la stessa coordinata X
  3. Conosco tutte le coordinate Y di tutti i punti, e valgono 1 per i punti in alto e 0 per i punti in basso.
  4. Le due rette si intersecano esattamente a metà e cioè a Y = 0.5
  5. Conosco la X del punto in cui si intersecano (nell'immagine segnata come G)

Sono sufficienti questi dati per trovarmi le coordinate X che mi servono?
In caso positivo, in che modo me le posso calcolare? Teorema dei seni facendo qualche ragionamento su che tipi di triangoli si formano? Ma io non ho angoli...

A voi la parola! Grazie mille in anticipo! :)

PS: G dovrebbe essere la media di Xa e Xb, giusto?

Ziosilvio 20-01-2012 10:45

Traccia una retta orizzontale che passa per il punto di intersezione la cui ascissa è G, e la verticale AC: chiama H il punto di intersezione delle due rette che hai tracciato, e K il punto in verde.
Allora i triangoli AHK ed ACD sono simili.

Da qui in poi dovrebbe essere facile...

luxorl 20-01-2012 11:41

Quote:

Originariamente inviato da Ziosilvio (Messaggio 36765891)
Traccia una retta orizzontale che passa per il punto di intersezione la cui ascissa è G, e la verticale AC: chiama H il punto di intersezione delle due rette che hai tracciato, e K il punto in verde.
Allora i triangoli AHK ed ACD sono simili.

Da qui in poi dovrebbe essere facile...

Il coefficiente angolare di quelle rette dovrebbe essere (se ho fatto bene i calcoli)

m1 = 1/2Xg
m2 = -m1

Conoscendo il coefficiente angolare risalgo agli angoli dei triangoli che mi hai fatto disegnare tu. A quel punto, sapendo gli angoli e la lunghezza di CH, applico il teorema dei seni e mi trovo il segmento HK. Da cui ovviamente mi ricavo le coordinate X del punto H

Corretto?

Ziosilvio 20-01-2012 13:52

Quote:

Originariamente inviato da luxorl (Messaggio 36766370)
Il coefficiente angolare di quelle rette dovrebbe essere (se ho fatto bene i calcoli)

m1 = 1/2Xg
m2 = -m1

Conoscendo il coefficiente angolare risalgo agli angoli dei triangoli che mi hai fatto disegnare tu. A quel punto, sapendo gli angoli e la lunghezza di CH, applico il teorema dei seni e mi trovo il segmento HK. Da cui ovviamente mi ricavo le coordinate X del punto H

Corretto?

Il coefficiente angolare di una retta non verticale che passa per i punti distinti (x1,y1) e (x2,y2) è (y1-y2)/(x1-x2).
Quindi, nel nostro caso abbiamo 1/2(xG-xC). Se C=(0,0) va bene come avevi scritto.

Se però non conosci xC le cose si fanno complicate.
È vero, in effetti, che xG è la media di xA ed xB. Ma è anche vero che non puoi ricostruire due numeri conoscendo solo la loro media.
Avresti bisogno o di uno dei due, o della loro differenza...

peppozzolo 22-01-2012 16:19

ragazzi scusate sembrera una domanda davero stupida ma capita spesso di imparare cose astruse e poi perdersi sui dettagli mi vergogno un po a chiederlo ma come trovo le soluzioni di un equazione del genere(quindi una specie di algoritmo generico) 2x^3+x^2-11x+5 con la quale non funziona nessun tipo di scomposizione da me conosciuta

grazie a chi mi sara d'aiuto

robertogl 22-01-2012 16:24

Quote:

Originariamente inviato da peppozzolo (Messaggio 36779292)
ragazzi scusate sembrera una domanda davero stupida ma capita spesso di imparare cose astruse e poi perdersi sui dettagli mi vergogno un po a chiederlo ma come trovo le soluzioni di un equazione del genere(quindi una specie di algoritmo generico) 2x^3+x^2-11x+5 con la quale non funziona nessun tipo di scomposizione da me conosciuta

grazie a chi mi sara d'aiuto

si può ridurre a (2x - 1)(x^2 + x - 5) se non sbaglio :)

peppozzolo 22-01-2012 16:27

bhe si c'ero arrivato anch'io con l'aiuto di wolfram ma piu che altro mi chiedevo come comportarmi un sua assenza :confused: come si fa a manina:D

robertogl 22-01-2012 16:40

Quote:

Originariamente inviato da peppozzolo (Messaggio 36779347)
bhe si c'ero arrivato anch'io con l'aiuto di wolfram ma piu che altro mi chiedevo come comportarmi un sua assenza :confused: come si fa a manina:D

usi Ruffini con termine che annulla il polinomio 1\2 :)

peppozzolo 22-01-2012 16:52

scusami lo so di essere insistente ma il mio dubbio e come si arriva a quel 1/2
grazie per l'attenzione :)

robertogl 22-01-2012 16:55

Quote:

Originariamente inviato da peppozzolo (Messaggio 36779519)
scusami lo so di essere insistente ma il mio dubbio e come si arriva a quel 1/2
grazie per l'attenzione :)

è il numero che annulla il polinomio di partenza 2x^3+x^2-11x+5. Dalle conoscenze che ho si va a tentativi.. ovviamente non provato 342, ma 1, -1, 2 e poi si vede se ci si allontana o avvicina dal risultato voluto, cioè appunto l'annullamento del polinomio :)

peppozzolo 22-01-2012 16:59

eh si ci avevo pensato ma onestamente speravo in qualche scappatoia ...mannaggia
grazie lo stesso comunque ;)
edit: quidni una cosa del genere potrebbe bloccarti un compito ?

ChristinaAemiliana 22-01-2012 20:35

Quote:

Originariamente inviato da peppozzolo (Messaggio 36779565)
eh si ci avevo pensato ma onestamente speravo in qualche scappatoia ...mannaggia
grazie lo stesso comunque ;)
edit: quidni una cosa del genere potrebbe bloccarti un compito ?

Premesso che nella maggior parte dei casi te la cavi a occhio raccogliendo opportunamente, per un caso come quello che hai portato esiste un teorema che ti permette di trovare le eventuali radici razionali di un'equazione polinomiale. QUI c'è la pagina di Wikipedia (da verificare perché non l'ho letta, comunque puoi sempre cercare altrove, meglio se su un libro). Questo dovrebbe rispondere alla tua domanda (se ho capito la tua domanda :stordita: ) e permetterti di procedere in maniera più metodica. :D

Per quanto riguarda il fatto di potersi bloccare in un compito, quello dipende ovviamente dal prof e da quanto possa sembrargli sensato inserire difficoltà del genere. La sorpresona può sempre esserci, a me capitò un compito di Macchine (!!!) compreso di equazione da risolvere numericamente, oltretutto pure di quelle terribili in cui un errore nella previsione del guess iniziale ti portava dritto alla soluzione priva di significato fisico :doh: (e non era nemmeno immediato accorgersene). Ma ecco, non è certo questa la norma, quindi non starei a preoccuparmi preventivamente...:D

Ziosilvio 22-01-2012 21:35

Quote:

Originariamente inviato da peppozzolo (Messaggio 36779292)
ragazzi scusate sembrera una domanda davero stupida ma capita spesso di imparare cose astruse e poi perdersi sui dettagli mi vergogno un po a chiederlo ma come trovo le soluzioni di un equazione del genere(quindi una specie di algoritmo generico) 2x^3+x^2-11x+5 con la quale non funziona nessun tipo di scomposizione da me conosciuta

grazie a chi mi sara d'aiuto

Si tratta di un polinomio di grado dispari a coefficienti reali, quindi ha (per un noto teorema di analisi) sicuramente una radice reale.
Per indovinare la quale si deve, di solito, andare un po' a naso...
Quote:

Originariamente inviato da ChristinaAemiliana (Messaggio 36780731)
Premesso che nella maggior parte dei casi te la cavi a occhio raccogliendo opportunamente, per un caso come quello che hai portato esiste un teorema che ti permette di trovare le eventuali radici razionali di un'equazione polinomiale. QUI c'è la pagina di Wikipedia (da verificare perché non l'ho letta, comunque puoi sempre cercare altrove, meglio se su un libro). Questo dovrebbe rispondere alla tua domanda (se ho capito la tua domanda :stordita: ) e permetterti di procedere in maniera più metodica. :D

Sapete una cosa? Non ci avevo mai pensato.
E sì che si vede subito, il motivo per cui deve essere vero...

stgww 23-01-2012 10:07

Algebra e relazioni
 
Ciao a tutti!
Ma come caspita si dimostra che m-n=7t è una relazione di equivalenza definita su Z ?
So che bisogna dimostrare che la relazione è
riflessiva ( aRa );
simmetrica (aRb -> bRa);
transitiva (aRb e bRc -> aRc);
Ma in pratica come faccio?

Grazie


Tutti gli orari sono GMT +1. Ora sono le: 17:58.

Powered by vBulletin® Version 3.6.4
Copyright ©2000 - 2024, Jelsoft Enterprises Ltd.
Hardware Upgrade S.r.l.